Euler's Totient Theorem Problem 1 Solution

Revision as of 20:17, 23 April 2021 by Borealbear (talk | contribs)

Problem

(BorealBear) Find the last two digits of $7^{81}-3^{81}$.

Solution

This is a direct application of Euler's Totient Theorem. Since $\phi(100)=40$, this reduces to $7^1-3^1\equiv \boxed{04}\pmod{100}$. -BorealBear